You are on page 1of 40

info@neoias.

com
info@neoias.com
9446331522 Download All PDF form here :- https://t.me/pdf4exams

INDIAN HISTORY AND


CULTURE –101 MOST
IMPORTANT TOPICS AND
QUESTIONS

 9446331522 www.neoias.com
9446331522 info@neoias.com

https://t.me/testseries4exams
Download All PDF form here :- https://t.me/pdf4exams
NEO IAS Most Important 101 History Model Questions from TPS 2018

NEO IAS  0484-4030104, 9446331522 Page 1


www.neoias.com |www.youtube.com/neoias|www.facebook.com/neoias | www.twitter.com/neoias

https://t.me/testseries4exams
Download All PDF form here :- https://t.me/pdf4exams
NEO IAS Most Important 101 History Model Questions from TPS 2018

1. Which of the following statements is/are not correct about Macaulay’s Minute 1835?
1. It opposed Downward Filtration Theory
2. It proposed Mass Education in vernacular languages
3. It proposed teaching of Western Sciences and Literature through Medium of English
4. It proposed teaching of Western Sciences and Literature through Medium of Sanskrit
Select the correct answer using the code given below.
(a) 1 and 4 only (b) 1, 2 and 4 only (c) 3 and 4 only (d) 1,2, 3 and 4

Answer (b)
Explanation
Lord Macaulay’s Minute, 1835: This famous minute settled the row in favor of
Anglicists – the limited government resources were to be devoted to teaching of western
sciences and literature through the medium of English language alone. Lord Macaulay
held the view that “Indian learning was inferior to European learning” – which was true
as far as physical and social sciences in the contemporary stage were concerned. The
government soon made English as the medium of instruction in its schools and colleges
instead of a large number of elementary schools, thus neglecting mass education.
The British planned to educate a small section of upper and middle classes, thus creating
a class. “Indian in blood and colour but English in tastes, morals, opinions and
intellect”. Who would act as interpreters between the government and the masses and
would enrich the vernaculars by which knowledge of western sciences and literature
would reach the masses. This was called the ‘downward filtration theory’.
Modern ideas, if not education, did filter down to the masses, though not on form desired
by the rulers, but through political parties, press, pamphlets, public platforms, etc.

2. With reference to the recommendations of Hunter education commission, consider the


following
1. It recommended the improvement of primary education
2. It recommended to impart primary education through English medium
3. It recommended for the spread of female education
Select the correct answer using the codes given below:
(a) 1 and 2 only (b) 2 and 3 only (c) 1 and 3 only (d) 1, 2, and 3

Answer (c)
Explanation
Hunter Education Commission, 1882-1883: This commission mostly confined its
recommendations to primary and secondary education. It
♦ Emphasized that state’s special care is required for extension and improvement of
primary education and that primary education should be imparted through
vernacular medium.
♦ Recommended transfer of control of primary education to newly set up district and
municipal boards.
♦ Recommended that secondary (High School) education should have two divisions –
♦ Literary-leading up to university
♦ Vocational-for commercial careers
♦ Drew attention to inadequate facilities for female education, especially outside
presidency towns and made recommendations for the spread of female education
3. The August Offer proposed by the then Viceroy of India, Lord Linlithgow, provided for
(a) The establishment of an advisory war council
(b) Transfer of power on universally agreed grounds
(c) Approval for the Nehru constitution

NEO IAS  0484-4030104, 9446331522 Page 2


www.neoias.com |www.youtube.com/neoias|www.facebook.com/neoias | www.twitter.com/neoias

https://t.me/testseries4exams
Download All PDF form here :- https://t.me/pdf4exams
NEO IAS Most Important 101 History Model Questions from TPS 2018

(d) Creation of Indian provincial government

Answer (a)
Explanation
AUGUST OFFER, 8 AUGUST 1940
The fall of France temporarily softened the attitude of congress in India. Britain was in
immediate danger of Nazi occupation. As the war was taking a menacing turn from
the allied point of view congress offered to cooperate in the war if transfer of authority in
India is done to an interim government. The government’s response was a statement
of the viceroy known as the august offer.
On 8 August 1940, early in the Battle of Britain, the Viceroy of India, Lord Linlithgow,
made the so-called August Offer. The following proposals were put in:
1. The establishment of an advisory war council
2. After the war, a representative Indian body would be set up to frame a constitution
for India.
3. Viceroy’s Executive Council would be expanded without delay.
4. The minorities were assured that the government would not transfer power “to any
system of government whose authority is directly denied by large and powerful
elements in Indian national life.”

4. With reference to ‘Governor Generals and associated events’, consider the following
statements:
1. Lord Wellesley led the British in the 4th Anglo-Mysore war.
2. Calcutta Medical College was founded by Lord William Bentinck.
3. Permanent Settlement Act was introduced by Lord Warren Hastings.
Which of the above statements is/are correct?
(a) 1 and 3 only (b) 1, 2 and 3 (c) 3 only (d) 1 and 2 only

Answer (d)
Explanation
The first and second statements are correct.
Permanent Settlement Act was introduced by Lord Cornwallis.
The Act was a long-term revenue contract with zamindars for a fixed amount of cash.
This Act ignored the ryots or tenants and their rights.

5. Which of the following statements about the Jallianwala Bagh Massacre is/are correct?
1. The crowd had gathered in the garden at Amritsar to peacefully protest the arrest
and deportation of their popular leaders, Dr.Satyapal and Dr. Kitchlu.
2. General Dyer was the viceroy of India at the time.
3. Rabindranath Tagore renounced his knighthood in protest against the massacre.
Select the answer from the codes below.
(a) 1, 2 and 3 (b) 2 only (c) 3 only (d) 1 and 3 only

Answer (d)
Explanation
JALLIANWALA BAGH MASSACRE (13 APRIL, 1919)
 The Jallianwala Bagh Massacre took place on 13 April 1919 and it remained a
turning point in the history of India’s freedom movement.
 In Punjab, there was an unprecedented support to the Rowlatt Satyagraha. Facing a
violent situation, the Government of Punjab handed over the administration to the
military authorities under General Dyer.
 He banned all public meetings and detained the political leaders.

NEO IAS  0484-4030104, 9446331522 Page 3


www.neoias.com |www.youtube.com/neoias|www.facebook.com/neoias | www.twitter.com/neoias

https://t.me/testseries4exams
Download All PDF form here :- https://t.me/pdf4exams
NEO IAS Most Important 101 History Model Questions from TPS 2018

 On 13th April, the Baisakhi day (harvest festival), a public meeting was organized
at the Jallianwala Bagh (garden).
 Dyer marched in and without any warning opened fire on the crowd. The firing
continued for about 10 to 15 minutes and it stopped only after the ammunition
exhausted. According to official report 379 people were killed and 1137 wounded
in the incident.
 The Jallianwala Bagh massacre (April 13, 1919) proved to be a key turning point in
Gandhi’s life. Gandhi was convinced that new tactics would be needed to obtain
social justice in India.
 Popular shock was expressed by the great poet and humanist Rabindranath
Tagore who renounced his knighthood in protest. C.F. Andrews, a friend of Tagore,
Mahatma Gandhi and the Nehrus, wrote to Mahadev Desai after a visit to Amritsar,
"It was a massacre, a butchery."

6. The Act which bifurcated provincial subjects into transferred and reserved for the first
time is.
(a) India Councils Act of 1861
(b) India Councils Act of 1892
(c) Government of India Act 1919
(d) Industrial Dispute Act of 1935
Answer (c)
Explanation
 The Government of India Act of 1919 provided a dual form of government (a
"dyarchy") for provinces.
 Matters of administration were first divided between the centre and the provinces
and then the provincial subjects were further bifurcated into transferred and
reserved subjects.
 The transferred subjects were to be administered by the Governor with the help of
ministers responsible to the legislative council composed mainly of elected members.

7. With reference to the Ilbert bill and related controversies, consider the following
statements
1. It was an attempt to abolish the racial prejudice from the Indian Penal Code.
2. According to the original bill, an Indian judge could try any non-European offenders.
3. It was to permit European judges the jurisdiction to try British offenders in criminal
cases at the District level.
4. It was passed without any amendment despite the vehement opposition of the British
resident in India.
Which of the statements given above is/are not correct?
(a) 1 and 3 only (b) 1 2 and 4 only (c) 3 and 4 only (d) 2 3 and 4 only

Answer (d)
Explanation
 The bill was introduced in 1883 by Viceroy Ripon, who actually desired to abolish
the racial prejudice from the Indian Penal Code.
 Ripon had proposed an amendment for existing laws in the country and to allow
Indian judges and magistrates the jurisdiction to try British offenders in criminal
cases at the District level.
 It was never allowed before. So naturally, the Europeans living in India looked it as a
Humiliation and the introduction of the bill led to intense opposition in Britain as
well as India (by the British residents).

NEO IAS  0484-4030104, 9446331522 Page 4


www.neoias.com |www.youtube.com/neoias|www.facebook.com/neoias | www.twitter.com/neoias

https://t.me/testseries4exams
Download All PDF form here :- https://t.me/pdf4exams
NEO IAS Most Important 101 History Model Questions from TPS 2018

 So it was withdrawn but was reintroduced and enacted in 1884 in a severely


compromised state. The amended bill had the provisions that the Europeans would
be conferred on European and Indian District Magistrates and Sessions Judges alike.
However, a defendant would in all cases have the right to claim trial by a jury of
which at least half the members must be European.
 Thus, this enactment held that Europeans criminals would be heard only by the
Indian Judges “helped by the European Judges”.
8. What is the historical sequence of the foundation of the following association?
1. Tattvabodhini Sabha
2. Prarthana Samaj
3. Ramakrishna Mission
4. Deccan Education Society
Select the correct answer from the codes given below:
(a) 3-2-1-4 (b) 4-3-2-1 (c) 1-2-4-3 (d) 1-3-2-4
Answer (c)
Explanation
 Tattvabodhini Sabha (1839)
 Prarthana Samaj 1867
 Deccan Education Society 1884
 Ramakrishna Mission 1897
9. Which of the following was/were the grievances of the sepoys leading to 1857 revolt?
1. Discrimination in payment and promotions
2. General Service Enlistment Act of 1846 restricted Indian recruits into army
3. British government ‘s order requiring the sepoys to serve overseas when required
4. Encouragement given to Christian missionaries by British army officers
Choose the correct answer using the codes below?
(a) 1, 2 and 3 b) 1, 3 and4 (c) 2, 3 and 4 (d) 1, 2, 3 and 4
Answer (b)
Explanation
The General Service Enlistment Act of 1856 required recruits to serve overseas if
ordered, a challenge to the castes who composed so much of the Bengal army.
10. The swaraj according to Gandhiji meant
1. the abandonment of the fear of death
2. goal that could be obtained easily and quickly with determination and courage.
3. it is sovereignty of the people based on pure moral authority
4. it is fruit of patience, perseverance, and intelligent appreciation of the environment.
Which of the statements given above is/are correct?
(a) 1 and 2 only (b) 2 and 3 only (c) 1, 3 and 4 only (d) 3 and 4 only
Answer (c)
Explanation
 Gandhi said that Politically, swaraj meant self-government and it means a
continuous effort to be independent of government control whether foreign or national.
 In other words, sovereignty of the people based on pure moral authority. Swaraj
is the abandonment of the fear of death.
 A nation which allows itself to be influenced by the fear of death cannot attain Swaraj,
and cannot retain it if some-how attained.
 It will be the fruit of patience, perseverance, ceaseless toil, courage and intelligent
appreciation of the environment.

NEO IAS  0484-4030104, 9446331522 Page 5


www.neoias.com |www.youtube.com/neoias|www.facebook.com/neoias | www.twitter.com/neoias

https://t.me/testseries4exams
Download All PDF form here :- https://t.me/pdf4exams
NEO IAS Most Important 101 History Model Questions from TPS 2018

11. With reference to the “Pabna Movement” consider the following statements
1. It was a agrarian unrest caused by oppressive practices of the Zamindars
2. it was held in the western parts of Maharashtra
3. The movement eventually resulted in large scale violence and massacre
4. Intellectuals like Bankim Chandra Chettarji, RC Dutt, supported the cause of
peasants
Which of the statements given above is/are not correct?
(a) 1 and 4 only (b) 2 and 3 only (c) 2, 3 and 4 only (d) 1, 3 and 4 only

Answer (b)
Explanation
 During the 1870s and 1880s, large parts of Eastern Bengal witnessed agrarian
unrest caused by oppressive practices of the Zamindars.
 The Zamindars resorted to enhanced rents beyond legal limits and prevented the
tenants from acquiring occupancy rights under Act X of 1859. Having had enough
of the oppressive regime, the peasants of Yusufshahi Pargana in Patna district
formed an agrarian league or combination to resist the demands of the Zamindars.
 The league organised a rent strike—the ryots refused to pay the enhanced rents,
challenging the Zamindars in the courts. Funds were raised by ryots to fight the court
cases. The struggles spread throughout Patna and to other districts of East Bengal.
 The main form of struggle was that of legal resistance; there was very little violence.
a number of young Indian intellectuals supported the peasants’ cause.
 These included Bankim Chandra Chatterjee, R.C. Dutt and the Indian Association
under Surendranath Banerjea.

12. Mahatma Gandhi’s severe criticism of the Congress leadership at the opening of the
Benaras Hindu University was focused on
(a) The congress strategy for opposing the government
(b) The congress accepting offices under the government
(c) Condemnation of Indian nationalism for being an elite phenomenon
(d) All of the above

Answer (c)

NEO IAS  0484-4030104, 9446331522 Page 6


www.neoias.com |www.youtube.com/neoias|www.facebook.com/neoias | www.twitter.com/neoias

https://t.me/testseries4exams
Download All PDF form here :- https://t.me/pdf4exams
NEO IAS Most Important 101 History Model Questions from TPS 2018

13. Which of the following statements about the Swadeshi movement in Bengal in 1905
is/are correct?
1. The celebration of Ganapati and Shivaji festivals, became a medium for Swadeshi
propaganda.
2. Swadesh Bandhab Samiti, the volunteer Organisation was set up by Ashwini Kumar
Dutt
3. Traditional folk theatre forms such as jatras were utilized to spread the swadeshi
ideas.
Select the correct answer using the code given below.
(a) 1 and 2 only (b) 2 and 3 only (c) 3 only (d) 1,2 and 3
Answer (d)
Explanation
 Corps of volunteers (or samitis as they were called) were one of major forms of mass
mobilization widely used by the Swadeshi Movement.
 The Swadesh Bandhab Samiti set up by Ashwini Kumar Dutt, a school teacher, in
Barisal was the most well-known volunteer organization of them all.
 Through the activities of this Samiti, whose 159 branches reached out to the remotest
corners of the district, Dutt was able to generate an unparalleled mass following
among the predominantly Muslim Peasantry of the region in 1905.
 The Swadeshi period also saw the creative use of traditional popular festivals and
melas as a means of reaching out to the masses.
 The Ganapati and Shivaji festivals, popularized by Tilak, became a medium for
Swadeshi propaganda not only in Western India but also in Bengal.
 Traditional folk theatre forms such as jatras i.e. extensively used in disseminating
the Swadeshi message in an intelligible form to vast sections of the people, many of
whom were being introduced to modern political ideas for the first time.

14. Consider the following statements


Asiatic Society of Bengal,
1. was founded by Sir William Jones
2. had the support and encouragement of then Governor General Wellesley
3. was set up to encourage Oriental studies
Which of the statements given above is/are correct?
(a) 1 only (b) 2 and 3 only (c) 1 and 3 only (d) 1,2 and 3
Answer (c)
Explanation
 Asiatic Society of Bengal, scholarly society founded on Jan. 15, 1784, by Sir William
Jones, a British lawyer and Orientalist, to encourage Oriental studies. The society
was founded shortly after his arrival.
 The Asiatic Society had the support and encouragement of Warren Hastings, the
governor-general (1772–85) of Bengal, though he declined its presidency.
 Until Jones’s death (1794) it was the vehicle for his ideas about the importance of
Hindu culture and learning and about the vital role of Sanskrit in the Aryan
languages. Indians were first admitted as members in 1829.

15. Consider the following statements:


1. The historic session of the Indian National Congress held at Belgaum in 1924 was
chaired by Gopal Krishna Gokhale
2. Sarojini Naidu was the first woman President of the Indian National Congress.
3. The decision that Swarajists would work in the council on behalf of the Congress
was endorsed in the Belgaum (1924) Session.

NEO IAS  0484-4030104, 9446331522 Page 7


www.neoias.com |www.youtube.com/neoias|www.facebook.com/neoias | www.twitter.com/neoias

https://t.me/testseries4exams
Download All PDF form here :- https://t.me/pdf4exams
NEO IAS Most Important 101 History Model Questions from TPS 2018

Which of the statements given above is/are correct?


(a) 1 and 3 (b) 2 and 3 (c) 1 only (d) 3 only

Answer (d)
Explanation
 The historic session of the Indian National Congress held at Belgaum in 1924 was
chaired by Mahatma Gandhi
 Annie Besant was the first woman President of the Indian National Congress.
 Sarojini Naidu was the first Indian woman President of the Indian National
Congress.
 The decision that Swarajists would work in the council on behalf of the Congress
was endorsed in the Belgaum (1924) Session
16. The demands of Young Bengal Movement included;
1. Female education
2. Universal adult suffrage
3. Protection of the ryots from oppressive zamindars
4. Self-rule for Indians
Select the correct answer using the code given below:
(a) 1, 2, 3 and 4 (b) 1 and 4 only (c) 1 and 3 only (d) 2 and 3 only

Answer (c)
Explanation
 The movement started by Derozio was called the Young Bengal Movement and his
followers were known as the Derozians.
 They condemned religious rites and the rituals, and pleaded for eradication of
social evils, female education and improvement in the condition of women.
 They carried on public agitation on public questions such as the revision of the
Company’s Charter, the freedom of the Press, better treatment for Indian labour
in British colonies abroad, trial by jury, protection of the ryots from oppressive
zamindars, and employment of Indians in the higher grades of government
services.
 But the extreme radical ideology of Derozians resulted in lack of support and gradual
decline of movement.

17. The members of the Constituent Assembly which drafted the Constitution of India were
(a) nominated by the British Parliament
(b) nominated by the Governor General
(c) elected by the legislative Assemblies of various provinces
(d) elected by the Indian National Congress and Muslim League
Answer (c)
Explanation
Elections of Members of Constituent Assembly
The constituent assembly was formed for the purpose of writing a constitution for
independent India. Members of Constituent Assembly were indirectly elected. During
British Era, India had provincial assemblies like the current legislative assemblies of
states. The members of the Constituent assembly were indirectly elected by the
members of the provincial assemblies by method of single transferable vote system of
proportional representations.

NEO IAS  0484-4030104, 9446331522 Page 8


www.neoias.com |www.youtube.com/neoias|www.facebook.com/neoias | www.twitter.com/neoias

https://t.me/testseries4exams
Download All PDF form here :- https://t.me/pdf4exams
NEO IAS Most Important 101 History Model Questions from TPS 2018

18. Which of the following statements about the Non-Cooperation Movement is/are correct?
1. Its spirit of defiance and unrest gave rise to local struggles like Awadh Kisan
Movement, Eka Movement (UP) etc
2. It was marked by the lack of labour unrest in places like Assam, Bengal and
Madras.
3. It was badly shaken by the Chauri Chaura incident in 1922 after which Gandhiji
decided to continue with the movement on a much smaller scale.
Select the correct answer using the code given below.
(a) 1 and 2 only (b) 2 and 3 only (c) 1 only (d) 2 only

Answer (c)
Explanation
 In Assam, strikes in tea plantations, steamer services, Assam-Bengal Railways
had been organised. J.M. Sengupta was a prominent leader in these strikes.
 The spirit of defiance and unrest gave rise to many local struggles such as Awadh
Kisan Movement (UP), Eka Movement (UP), Mappila Revolt (Malabar) and the
Sikh agitation for the removal of mahants in Punjab.
 It was badly shaken by the Chauri Chaura incident in 1922 after which Gandhiji
decided to suspend the movement.

19. With reference to the land revenue policies of British, consider the following statements
1. Under Zamindari system,Zamindars were regularised as landlords and were given
power to evict cultivator
2. Mahalwari system was introduced during the period of Warren Hastings
3. Under Ryotwari system peasants were in direct contact with state in paying land
revenue
Select the correct answer using the code below
(a) 1 and 3 only (b) 2 and 3 only (c) 3 only (d) 1, 2 and 3

Answer (a)
Explanation
 Under Zamindari system, Zamindars were regularised as landlords and were given
power to evict cultivator.
 Mahalwari system was introduced during the period of William Bentick
 Under Ryotwari system peasants were in direct contact with state in paying land
revenue

20. Which of the following statements is correct about the battle of Plassey – 1757?
(a) It made the East India Company a dependent of the Mughal ruler.
(b) It exposed the superiority of the Indian military skills and arms.
(c) It paved the way for the British mastery of Bengal and eventually the whole of India.
(d) It made East India Company to introduce the subsidiary alliance in India.

Answer(c)
Explanation
 It was not Plassey but Buxar battle made the Mughal ruler a dependent of the East
India Company The battle of Plassey was not important from the military view
point.
 It was a mere conflict. No military superiority was shown by the English army.
 The battle of Plassey followed the subsequent plunder of Bengal as Bengal was
placed at the disposal of the English vast resources.

NEO IAS  0484-4030104, 9446331522 Page 9


www.neoias.com |www.youtube.com/neoias|www.facebook.com/neoias | www.twitter.com/neoias

https://t.me/testseries4exams
Download All PDF form here :- https://t.me/pdf4exams
NEO IAS Most Important 101 History Model Questions from TPS 2018

 The vast resources of Bengal helped the Britishers to conquer the Deccan and
extend their influence over North India.
 The Battle of Plassey ushered in a new era in the history of India.
 It was a turning point not only in the history of Bengal but also in the history
of whole of India.
 It has, therefore, been rightly remarked that the Battle of Plassey marked the end
of one epoch and the beginning of another.

21. Which of the following is a correct statement about Quit India Movement?
1. It became popular by the name August Revolution.
2. The Cripps mission proposals ended the Quit India movement
3. The Peasants of all strata were the heart of the movement.
4. The Communist Party of India and All India Trade Union Congress conducted hartals
during the Quit India Movement.
Select the correct answer using the codes given below:
(a) 1 and 2 only (b) 2, 3 and 4 only (c) 1 and 3 only (d) 3 and 4 only

Answer (c)
Explanation
 ‘Quit India,’ ‘Bharat Choro’. This simple but powerful slogan launched “the legendary
struggle which also became famous by the name of the ‘August Revolution.’
 The failure of the Cripps Mission in April 1942 was one of the main reasons for
the outbreak of Quit India movement. While it is true that Muslim mass
participation in the Quit India movement was not high, yet it is also true that even
Muslim League supporters not act as informers.
 Also, there was a total absence of any communal clashes, a sure sign that though
the movement may not have aroused much support from among the majority of the
Muslim masses, it did not arouse their hostility either.
 Peasants of all strata, well-to-do as well as poor, were the heart of the movement.
 Peasants of all strata, well-to-do as well as poor, were the heart of the movement
especially in East U.P. and Bihar, Midnapur in Bengal, Satara in Maharashtra, but
also in other parts including Andhra, Gujarat and ‘Kerala. Many small zamindars also
participated especially in U.P. and Bihar.

22. Which one of the following was not a recommendation of Strachey Commission?
(a) A famine fund should be set up
(b) A famine code should be formulated.
(c) Irrigation facilities should be developed
(d) Agriculture should come under reserved subjects

Answer (d)
Explanation
The Government appointed Richard Strachey Commission (famine) in 1880.The
Commission recommended
1. A famine code should be formulated.
2. Irrigation facilities should be developed.
3. Collection of land revenue should be suspended immediately during famines
and land revenue should be remitted.
4. Data should be collected about the conditions of Indian peasantry and
agriculture.
5. A famine fund should be set up.

NEO IAS  0484-4030104, 9446331522 Page 10


www.neoias.com |www.youtube.com/neoias|www.facebook.com/neoias | www.twitter.com/neoias

https://t.me/testseries4exams
Download All PDF form here :- https://t.me/pdf4exams
NEO IAS Most Important 101 History Model Questions from TPS 2018

23. Which one of the following was a result of British Colonial rule and their economic policies
in India?
(a) Britain became a supplier of raw materials to India.
(b) India became a manufacturing hub.
(c) British goods got discouraged in Indian market.
(d) Indian handicraft industries ruined.

Answer (d)
Explanation
With the introduction of colonial rule in India, the economic policies followed by British
were more concerned about protection and promotion of the economic interests of their
own country rather than development of Indian economy under British rule.
Prior to British colonial rule, India was well known for its handicraft industries dealing
with cotton and silk textiles, metal and precious stone works, etc. These products
manufactured in India had a worldwide market due to fine quality of material used and
high standards of craftsmanship employed in the exported products.
The policies followed by the company rule brought about a fundamental change in the
structure of the Indian economy, transforming India into a supplier of raw materials and
a consumer of finished industrial products from Britain. Consequently, Indian domestic
industries ruined.

24. Which of the following statement/ statements is/are correct about the Paika Rebellion of
1817?
1. It was an armed rebellion against British colonialism.
2. Paikas were the traditional land-owning militia of Odisha.
3. They revolted against the British as the British banned their rituals.
4. Bakshi Jagabandhu Bidyadhar, the military chief of the King of Khurda, led the army
of Paikas.
Select the correct answer using the codes given below.
(a) 1 and 3 only (b) 1 and 4 only (c) 1, 2 and 4 only (d) 2 and 3 only

Answer (c)
Explanation
Paika Rebellion of 1817 is an armed rebellion against British colonialism. The oppressive
agrarian cum land reforms of British forced the paikas of Odisha rose in rebellion. Bakshi
Jagabandhu Bidyadhar, the military chief of the King of Khurda, led the army of Paikas
and forced the British to retreat. This came to be known as Paika Bidroh (Paika rebellion).
The Paikas were the traditional land-owning militia of Odisha and served as warriors.
The main cause of the revolution was the takeover of the rent-free land that had been
given to the Paiks for their military service to the Kingdom of Khurda.

25. The Cabinet Mission proposals did not include


(a) the formation of a Union of India
(b) setting up an Interim Government
(c) formation of the constituent assembly
(d) a strong centre with viceroy as the head

Answer (d)
Explanation
The Cabinet Mission proposals included Provisions for three groups of provinces to
possess

NEO IAS  0484-4030104, 9446331522 Page 11


www.neoias.com |www.youtube.com/neoias|www.facebook.com/neoias | www.twitter.com/neoias

https://t.me/testseries4exams
Download All PDF form here :- https://t.me/pdf4exams
NEO IAS Most Important 101 History Model Questions from TPS 2018

their separate constitutions. It proposed the formation of a Union of India, comprising


both the British India and the Princely States. Union would remain in charge of only
foreign affairs, defence and communications leaving the residuary powers to be vested
in the provinces. It proposed setting up an Interim Government, which would remain
in office till a new government was elected on the basis of the new Constitution framed
by the Constituent Assembly. Rejection of the demand for a full-fledged Pakistan. It also
provided for formation of the constituent assembly on democratic principle of
population.

26. With reference to the Wavell plan proposals, consider the following statements:
1. It proposed the Central Executive Council to be Indianised.
2. All portfolios including defence would be held by the Indian Members.
3. It is also known as Breakdown Plan and was not accepted by the British.
Which of the statement/ statements given above is/are correct?
(a) 1 and 3 only (b) 2 only (c) 3 only (d) 1 and 2 only

Answer (c)
Explanation
With the exception of the Governor-General and the Commander-in-chief, all other
member of the Executive Council would be nominated from amongst leaders of Indian
Political life. All portfolios except the Defence would be held by the Indian Members.
Wavell Plan is also known as Breakdown Plan and was not accepted by the British, for
whom, leaving without a universally agreed agreement was dishonorable. It also said that
in case of a disagreement, the British should withdraw to the 6 Pakistan Provinces,
and leaving the Congress to deal with rest of India.

27. Consider the following statements regarding Home Rule Movement:


1. It aimed to attain self-government within the British empire.
2. It was able to combine the moderates, extremists, and revolutionary terrorists.
3. It dropped the idea of mendicancy of the moderates but continued their concept of
patriotism.
4. Tilak made the concept of Home Rule, the popular catchword through the medium of
New India and The Commonweal.
Which of the statements given above is/are correct?
(a) 1, 2 and 3 only (b) 2 and 4 only (c) 1, 3 and 4 only (d) 2 only

Answer (a)
Explanation
In September 1916, Annie Besant started the Home Rule League as an independent
political organization. A few months earlier (April) Tilak had established an Indian
Home Rule League with the object of attaining Home Rule or self-government. Home
Rule Movement was able to combine and balance all the three trends viz. moderates,
extremists, and revolutionary terrorists. Home Rule Movement continued extremist’s
idea of passive resistance. Home Rule Movement dropped the idea of mendicancy of
the moderates but continued their concept of patriotism. Through his writings in the
Mahratta and Kesari, Tilak made the concept of Home Rule the popular catchword.
Annie carried on her campaign through the medium of New India and The
Commonweal.

28. Which one among the following was not a part of Delhi Pact of 1931?
(a) Return of all lands not yet sold to third parties.
(b) Remission of all fines not yet collected.

NEO IAS  0484-4030104, 9446331522 Page 12


www.neoias.com |www.youtube.com/neoias|www.facebook.com/neoias | www.twitter.com/neoias

https://t.me/testseries4exams
Download All PDF form here :- https://t.me/pdf4exams
NEO IAS Most Important 101 History Model Questions from TPS 2018

(c) Right to peaceful and non-aggressive picketing.


(d) Commutation of Bhagat Singh’s death sentence

Answer (d)
Explanation
Gandhiji was authorised to negotiate with the Viceroy Lord Irwin in relation to deciding
congress participation in second round table conference. As a result of their
discussions, a pact was signed between the viceroy, representing the British Indian
Government, and Gandhi, representing the Indian people, in Delhi in March, 1931.
This Delhi Pact, also known as the Gandhi-Irwin Pact, placed the Congress on an equal
footing with the Government.
IRWIN ON BEHALF OF THE GOVERNMENT AGREED ON
1. Immediate release of all political prisoners not convicted of violence;
2. Remission of all fines not yet collected;
3. Return of all lands not yet sold to third parties;
4. Lenient treatment to those government servants who had resigned;
5. Right to make salt in coastal villages for personal consumption (not for sale);
6. Right to peaceful and non-aggressive picketing; and
7. Withdrawal of emergency ordinances.
TWO DEMANDS OF GANDHI IGNORED
1. Public inquiry into police excesses, and
2. Commutation of Bhagat Singh and his comrades’ death sentence to life sentence.
What is important here- Gandhi-Irwin Pact

29. With reference to INA trial agitation, consider the following statements:
1. All India Muslim League supported it.
2. Communist Party of India sided with British.
3. It demanded release of Bhagat Singh and Nehru.
4. It lacked mass support at any stage.
Which of the statements given above is/are correct?
(a) 1 only (b) 2 only (c) 1 and 4 only (d) 2, 3 and 4 only

NEO IAS  0484-4030104, 9446331522 Page 13


www.neoias.com |www.youtube.com/neoias|www.facebook.com/neoias | www.twitter.com/neoias

https://t.me/testseries4exams
Download All PDF form here :- https://t.me/pdf4exams
NEO IAS Most Important 101 History Model Questions from TPS 2018

Answer (a)
Explanation
The campaign had a wide geographical reach and witnessed the participation of
diverse social groups and political parties. Those who supported the INA cause in
varying degrees, apart from the Congress, included the Muslim League, Communist
Party, Unionists, Akalis, Justice Party, Ahrars in Rawalpindi, Rashtriya
Swayamsevak Sangh, Hindu Mahasabha and the Sikh League. It demanded release of
all INA soldiers, officers who face the trial by the British.

30. Which of the following statements with regards to the Civil Disobedience movement is/are
correct?
1. The movement received no response from business groups and peasantry.
2. The movement coincided with Ramosi uprisings in Maharashtra.
3. The movement was marked by the mass participation of women and students.
Select the correct answer using the code given below.
(a) 1 only (b) 1 and 2 only (c) 3 only (d) 1, 2 and 3

Answer (c)
Explanation
The movement received massive response from business groups and peasantry since they
realized British government policies never protect or support the interests of Indian
business class or peasantry. The movement was marked by the mass participation of
women (perhaps, the largest scale women participation) and students. The movement did
not coincide with Ramosi uprisings. The Ramosis, the hill tribes of the Western Ghats,
had not reconciled to British rule and the British pattern of administration. They rose
under Chittur Singh in 1822 and plundered the country around Satara. Again, there
were eruptions in 1825-26 and the disturbances continued till 1829.The disturbance
occurred again in 1839 over deposition and banishment of Raja Pratap Singh of Satara,
and disturbances erupted in 1840-41 also. Finally, a superior British force restored order
in the area.

31. Which of the following statement/ statements about the Champaran Satyagraha is/are
correct?
1. It was due to forced opium cultivation.
2. Rajendra Prasad invited Gandhiji to Champaran.
3. The Champaran satyagraha resulted in abolition of teenkathiya.
Select the correct answer using the code given below.
(a) 1, 2 and 3 only (b) 2 only (c) 1 and 2 only (d) 3 only

Answer (d)
Explanation
 The Champaran Movement of Bihar in 1917 was a result of the long colonial
oppressive "teenkathiya system" where the planters forced cultivators to grow
indigo (a blue dye) in 3/20th parts of their lands.
 It addressed the economic demands of the peasants. Champaran satyagraha also
featured for the first time that bridges had been built between the peasants and the
other sections, especially the middle-class intelligentsia, such as Rajendra Prasad,
J.B.Kriplani etc.
 It was Rajkumar Shukla who invited Gandhiji to visit Champaran and inquire about
problems of its cultivators.
 The Champaran Satyagraha resulted in abolition of teenkathiya.

NEO IAS  0484-4030104, 9446331522 Page 14


www.neoias.com |www.youtube.com/neoias|www.facebook.com/neoias | www.twitter.com/neoias

https://t.me/testseries4exams
Download All PDF form here :- https://t.me/pdf4exams
NEO IAS Most Important 101 History Model Questions from TPS 2018

32. Which one of the following is a feature of the Nehru Report of 1928?
(a) Federal form of government with residuary powers vested in the provinces
(b) Federal form of government with residuary powers vested in the centre
(c) Rejection of joint electorates
(d) Rejection of dominion status

Answer (b)
Explanation
IMPORTANT RECOMMENDATIONS OF NEHRU REPORT
The Nehru report stated that the `next immediate step for India must be ‘dominion
status’,
the Nehru report was approved by the congress at Calcutta in December 1928.Other
important recommendations included rejection of separate electorates which had been
the basis of constitutional reforms so far; instead, a demand for joint electorates with
reservation of seats for Muslims at the centre and in provinces where they were in
minority.
1. There should be federal form of government with residuary powers vested in the
centre. It included a description of the machinery of government including a proposal
for the creation of a Supreme Court and a suggestion that the provinces should be
linguistically determined. It recommended for approval of nineteen fundamental
rights including equal rights for women, right to form unions, and universal
adult suffrage. Another proposal was full protection to cultural and religious
interests of Muslims.

33. Which of the following statements is/are correct about the final phase of RIN mutiny?
1. Karachi became a major center of the mutiny.
2. Sympathetic token strikes took place in military establishments in Madras.
3. Delhi, Cochin, and Jamnagar shore establishments were not affected by the mutiny.
4. Students didn’t respond unlike other national movements.
Select the correct answer using the code given below.
(a) 1, 2, 3 and 4 (b) 1 and 2 only (c) 1, 2 and 3 only (d) 3 and 4 only

Answer (b)
Explanation
The third phase was characterized by a display of solidarity by people in other parts of
the county. Students boycotted classes, hartals and processions were organized to
express sympathy with the students and ratings and to condemn official repression. In
the RIN revolt, Karachi was a major centre, second only to Bombay. The news reached
Karachi on 19 February, upon which the HMIS Hindustan along with one more ship and
three shore establishments, went on a lightning strike. Sympathetic token strikes took
place in military establishments in Madras. Vishakhapatnam. Calcutta, Delhi, Cochin,
Jamnagar, the Andamans, Bahrain and Aden Seventy-eight ships and 20 shore
establishments, involving 20,000 ratings, were affected.

34. With reference to the Lucknow session of Indian National Congress – 1916, which of the
statements given below is/are correct?
1. It strengthened the ideological alienation of Extremists.
2. The alliance between Congress and Muslim League was made in this session.
3. The Resolution condemning the Arms Act and Press Act were passed in this session.
4. Congress rejected the separate electorate proposal of Muslim League.
Select the correct answer using the codes given below.
(a) 2, 3 and 4 only (b) 1, 3 and 4 only (c) 1, 2 and 3 only (d) 2 and 3 only

NEO IAS  0484-4030104, 9446331522 Page 15


www.neoias.com |www.youtube.com/neoias|www.facebook.com/neoias | www.twitter.com/neoias

https://t.me/testseries4exams
Download All PDF form here :- https://t.me/pdf4exams
NEO IAS Most Important 101 History Model Questions from TPS 2018

Answer (d)
Explanation
LUCKNOW SESSION OF 1916 AND LUCKNOW PACT
The 31st Session of the Congress was held at Lucknow in 1916. It was presided over by
the Ambica Charan Majumdar who was a prominent lawyer and was actively associated
with the Congress since its birth. After a lapse of about 10 years both the Moderates and
Extremists were united again which was a good sign for the national movement. In
this session the Congress and the Muslim League came closer to each other and they
signed the historic Lucknow Pact. A joint Reform Scheme was sent to the Viceroy.
They decided to make a united demand for self-government. They were to join their
hands in asking the Government that a majority of the members of the Legislative
Councils to be elected. They were to ask the Government that the Legislative Councils be
invested with wider powers than before. They would make a common demand that at
least half the seats in the Viceroy's Executive Council be filled with Indians. Thus,
this session of 1916 cemented the friendship between the Congress and the Muslim
League and promoted goodwill between the Hindus and the Muslims. Resolution
condemning the Arms Act and Press Act were passed which had virtually reduced the
people and the press to a condition of absolute helplessness.
What is important here- Important Congress sessions

35. Consider the following statements:


1. Rukhmabai Raut condemned the Age of Consent Act 1891
2. She wrote under the pseudonym, A Hindu Lady
3. She was one of the founding members of the Brahma Samaj.
Which of the statements given above is/ are correct?
(a) 1, 2 and 3 (b) 2 only (c) 3 only (d) 1 and 3 only

Answer (c)
Explanation
Rukhmabai Raut, who was the first women doctor to practice medicine in British India
for a long term and she staunchly opposed child marriage. Raut became the major cause
behind the enactment of Age of Consent Act in 1891. She wrote many influencing
letters under the pseudonym A Hindu Lady.

36. During the Indian freedom struggle, a series of meetings referred to as Round Table
Conference were held primarily to discuss the :
(a) Grant of Independence
(b) Resolution of Religious Conflict
(c) Educational and Social Reforms
(d) Structure of Judicial system

37. Consider the following statements about the Morley-Minto reforms of 1909:
1. A provision for appointment of an Indian to Viceroy’s executive council was made for
the first time.
2. Surendra Nath Banerjee became the first Indian appointed to the viceroy’s executive
council and he was given law portfolio.
3. They contained provisions that ensured that British officials retained their majority in
the Imperial Legislative Council.
Which of the statements given above is/are correct?
(a) 1 only (b) 2 and 3 only (c) 1 and 3 only (d) 1, 2 and 3

NEO IAS  0484-4030104, 9446331522 Page 16


www.neoias.com |www.youtube.com/neoias|www.facebook.com/neoias | www.twitter.com/neoias

https://t.me/testseries4exams
Download All PDF form here :- https://t.me/pdf4exams
NEO IAS Most Important 101 History Model Questions from TPS 2018

Answer (c)
Explanation
THE INDIAN COUNCILS ACT OF 1909
The Act provided for the expansion of the Legislative Councils at both the levels, central
as well as provincial. It maintained the majority of official members in the Central
Legislative Council. There were four categories of members i.e. ex-officio members,
nominated officials, nominated non-officials and elected members. It provided for non-
official majority in the Provincial Legislatures but then, the combined strength of official
and nominated non-official members out-numbered the elected members. INDIAN TO
VICEROY’S EXECUTIVE COUNCIL
A provision for appointment of an Indian to Viceroy’s executive council was made for the
first time. Satyendra Prakash Sinha became the first Indian appointed to the viceroy’s
executive council and he was given law portfolio.

38. Which one among the following is correct about the Doctrine of Lapse?
(a) It did not allow the Indian rulers to adopt any heir
(b) It did not allow an adopted heir to rule a state after the death of the ruler
(c) It made the annexation of Indian state compulsory after the death of a ruler
(d) It made the annexation of India state compulsory if the adoption of heir had not been
approved by the British authorities

39. With reference to the major findings from Indus sites consider the following statements
1. Fossils of horse bones are found from Surkotada
2. A steatite male head is found from Harappa
3. Lothal is known for its unique water management system
Select the correct statement/statements using the codes given below.
(a) 1 only (b) 2 only (c) 2 and 3 only (d) 1, 2 and 3

Answer (a)
Explanation
SITE LOCATION RIVER
MAJOR FINDINGS
Gateway city, fortified wall,
Harappa Punjab
workmen’s quarter, granaries,
1 Ram Sahini province, Ravi
sandstone statues, coffin burial
(1921-1923) Pakistan
etc
Great Bath, Great Granary,
Collegiate Building and
Mohenjodaro
Sindh Assembly Hall. A steatite male
2 R.DBanerji Indus
Pakistan head, the bronze dancing girl and
(1922)
complex toys. Piece of woven
cotton. pashupati seal
Fire althars, camel bones, Mud-
Kalibangan
3 Hanumangarh Ghaggar brick fortification, discovery of a
Amalnand Ghosh
Rajasthan plough field,
Koti-Diji (Pre- Statue of mother goddess and
4 Sindh Indus
Harappan) bull
Brick or Stone fortification,
5 Chanhu-Daro Sindh Indus
multiple factories, inkpot
Saraswat Pre Harappan and Harappan sites;
8 Banwali Hissar Haryana
i remain of streets and drains;

NEO IAS  0484-4030104, 9446331522 Page 17


www.neoias.com |www.youtube.com/neoias|www.facebook.com/neoias | www.twitter.com/neoias

https://t.me/testseries4exams
Download All PDF form here :- https://t.me/pdf4exams
NEO IAS Most Important 101 History Model Questions from TPS 2018

traces of barley. Terracotta


plough
Trade point between Harappa and
Suktagendor(Bal
10 Pakistan Dast Babylon; situated on natural rock;
uchistan)
western most site.
A coastal site, dockyard built with
Lothal
11 Gujarat Bhogwa brick, rice husk, evidence of
Ahemedabad
cotton, bead factory, Iranian Seal,
Gujarat Flourishing urban centres, bead
12 Surkotada
Rann of Kutch industry, fossil remains of horse.
Several cultural stages; three parts
Gujarat of city, unique water management,
13 Dholavira
Rann of Kutch largest Harappan inscription, use
of fire-altars.

40. Which of the following characterizes/ characterize the early Vedic age
1. The lowest unit of the society was the patriarchal family.
2. The people practiced limited cultivation/ agriculture.
3. Sabha and Vidatha were the most important assemblies
Select the correct statement/statements using the codes given below.
(a) 1 and 2 only (b) 2 only (c) 1, 2 and 3 only (d) None of the above

Answer (a)
Explanation
The lowest unit of the Rig-Vedic society was the patriarchal family. The chief social
unit of the Aryans was known as jana. The chief of this unit was the political leader called
rajan. The main function of the chief was to protect the jana and cattle from the
enemies. He was helped in his task by the tribal assemblies called sabha, samiti,
vidatha, gana and parishad. Out of these sabha and samiti were the most important
assemblies. All aspects of life were discussed in these assemblies. These may include
wars, distribution of the spoils of wars, judicial and religious activities etc.

41. With reference to Nyaya philosophy, consider the following statements


1. Salvation is attained through destruction of soul
2. salvation can be attained through the acquisition of valid knowledge.
3. It was the earliest school of philosophy
Which of the statements given above is/are correct?
(a) 1 and 2 only (b) 2 only (c) 3 only (d) 1, 2 and 3

Answer (b)
Explanation
Nyaya is one of the six orthodox schools of philosophy. This school's most significant
contributions to Indian philosophy was systematic development of the theory of logic.
According to it salvation can be attained through the acquisition of valid knowledge.
what is more important, the truth of a proposition or statement can be tested through
inference, hearing and analogy. An example of how they use logic is given below
1. There is fire in the mountain
2. Because it emits smoke
3. Whatever emits smoke contains fire as the hearth (Kitchen)

NEO IAS  0484-4030104, 9446331522 Page 18


www.neoias.com |www.youtube.com/neoias|www.facebook.com/neoias | www.twitter.com/neoias

https://t.me/testseries4exams
Download All PDF form here :- https://t.me/pdf4exams
NEO IAS Most Important 101 History Model Questions from TPS 2018

The stress laid on the use of logic influenced Indian scholars who look to systematic
thinking and reasoning. Samkhya is one of the most prominent and one of the oldest
of Indian philosophies.

42. With reference to the semi-classical music form Thumri, consider the following
statements
1.The thumri music originated in Karnataka
2. The text is romantic and devotional in nature
3. Its language is a dialect of Hindi called suri bhasha
Which of the statements given above is/ are correct?
(a) 1, and 2 only (b) 2 only (c) 3 only (d) 1, 2 and 3 only

Answer (b)
Explanation
Indian Hindustani Classical Music has various divisions like Khyaal, Thumari, Tappa,
Tarana, etc. The thumri is based on the romantic-devotional literature inspired by the
bhakti movement. The text is usually derived from the Radha-Krishna theme and is of
primary importance. The semi-classical music form “thumri” is known to have derived its
name from thumakna. Loosely translated the word means, “dance-like movements”.
Thumri was about mild eroticism and dramatic gestures and was the invention of
courtesans. The text is romantic and devotional in nature, and usually revolves around
a girl's love for Krishna. The language is a dialect of Hindi called Brij bhasha. This style
is characterized by a greater flexibility with the rag. It arose in popularity during the 19th
century.

43. Which of the following statement/statements is/are correct about first Buddhist council?
1. It was convened to scrap Madhyamika philosophy
2. It was held at Rajgriha, in the Sattapani Cave
3. It was held under the patronage of king Mahakasyapa
4. The chairman of the council was Sabakami
Select the correct statement/statements using the codes given below.
(a) 1 only (b) 2 only (c) 1, 2 and 3 only (d) 3 and 4 only

Answer (b)
Explanation
FIRST BUDDHIST COUNCIL
First Buddhist council was held soon after the mahaparinirvana of the Buddha, around
483 BC under the patronage of king Ajatshatru with the monk Mahakasyapa presiding,
at Rajgriha, in the Sattapani Cave. The idea was to preserve and codify Buddha’s
teachings (Sutta) and rules for disciples (Vinaya). Ananda, one of the great disciples of
Buddha recited Suttas and Upali, another disciple recited Vinaya.
44. Which of the following is correctly paired?
(a) Kaira Movement –Karnataka
(b) Namsudra Movement –Bengal
(c) Justice Movement-Orissa
(d) Lingayat Movement – Madhya Pradesh

Answer (b)
Explanation
(a) Justice Movement-Tamil nadu
(b) Kaira Movement –Gujarat
(c) Lingayat Movement – Karnataka
NEO IAS  0484-4030104, 9446331522 Page 19
www.neoias.com |www.youtube.com/neoias|www.facebook.com/neoias | www.twitter.com/neoias

https://t.me/testseries4exams
Download All PDF form here :- https://t.me/pdf4exams
NEO IAS Most Important 101 History Model Questions from TPS 2018

(d) Namsudra Movement –Bengal


 The Namasudras were a "large non-Aryan caste of eastern Bengal, mainly engaged
in boating and cultivation. Later claimed the "more respectable title of 'Namasudra'
and Brahmin status".
 According to Sekhar Bandyopadhyay, the Dalit of Bengal became involved in the
Partition movement, and that the "two most important communities, who dominated
dalit politics in the province, were the Namasudras and the Rajbanshis".

45. Cornelia Sorabji


1. was first Indian woman to study law at Oxford University
2. was the second woman president of INC
3. took up the cause of purdanashins
Which of the statements given above is are correct?
(a) 1 and 2 only (b) 2 only (c) 2 and 3 only (d) 1 and 3 only

Answer (d)
Explanation
 Cornelia Sorabji was the first woman to practice law in India and Britain
 She was the first woman to graduate from Bombay University and she was the
first women to study in any British university.
 She took up the cause of purdanashins, women who spent their lives behind the
veil, forbidden to interact with men outside their families.
 Sorabji helped widowed purdahnashins get their rightful share of their deceased
husband's property and also encouraged them to pursue education and secure
employment.
46. Consider the following statements
1. Allahabad mill strike was due to passage of Chelmsford reforms
2. It was triggered by withdrawal of plague-bonus
3. Anasuya Sarabhai opposed the strike
Which of the statements given above is/are correct?
(a) 1 only (b) 2 only (c) 1 and 2 only (d) 1 2 and 3

Answer (b)
Explanation
 In the middle of the Kheda satyagraha, Gandhi also got involved in the Ahmedabad
textile mill strike of Febrauary-March 1918.
 The immediate reason for Ahmedabad textile mill strike was the ‘withdrawal of
plague-bonus’, which was being given to prevent workers from leaving the city in the
face of mounting plague-related deaths.
 Anasuya Sarabhai worked for the welfare of disempowered women, and focused on
mill workers who had to endure thirty-six hour long shifts.
 She negotiated with mill owners for better work conditions, and even though her
brother was a mill owner, she helped organise the first strike of Ahmedabad weavers
in a demand for higher wages.
 Montague Chelmsford reforms was passed and took final shape in the form of
Government of India Act 1919.
47. Padmavat was
(a) written in Bengali language
(b) originally authored by Persian poet Amir khussru
(c) a 16th century historical work by Malik Faisie
(d) a fictional epic poetry in Awadhi language
NEO IAS  0484-4030104, 9446331522 Page 20
www.neoias.com |www.youtube.com/neoias|www.facebook.com/neoias | www.twitter.com/neoias

https://t.me/testseries4exams
Download All PDF form here :- https://t.me/pdf4exams
NEO IAS Most Important 101 History Model Questions from TPS 2018

Answer (d)
Explanation
The Padmavat was a 16th century fictional epic poetry, written in Awadhi language
by the Sufi poet, Malik Muhammad Jayasi.

48. Which one of the following political dramas of ancient Gupta period dealt with story of
young Brahmin cum merchant, who falls in love with a wealthy courtesan,
Vasantasena?
(a) Mrichchhakatika (b) Kaumudimahotsava
(c) Malavikagnimitra (d) Devichandraguptam

Answer (a)
Explanation
Mṛichchakaṭika (The Little Clay Cart) is a sanskrit drama attributed to Sudraka, an
ancient playwright generally thought to have lived sometime between the third century
BC and the fifth century AD whom the prologue identifies as a Kshatriya king and a
devotee of Siva who lived for 100 years. The central story is that of noble but
impoverished young Brahmin, Charudatta who falls in love with a wealthy courtesan
Vasantasena. Sanskrit texts and inscriptions used the term vanik to designate
merchants. While trade was defined as an occupation for Vaishyas in the Shastras, a
more complex situation is evident in plays such as the Mrichchhakatika written by
Shudraka (c. fourth century CE), Here, the hero Charudatta was described as both a
Brahmana and a sarthavaha or merchant

49. With reference to Sangam literature, consider the following statements


1. The widowhood was miserable in Sangam society
2. The practice of Sati in the higher varna was absent
3. The Hero Stone worship was significant in this period
4. People strictly adhered to monotheism
Which of the statements given above is/are correct?
(a) 1, 2 and 4 only (b) 3 and 4 only (c) 1 and 3 only (d) 1, 2, 3 and 4

NEO IAS  0484-4030104, 9446331522 Page 21


www.neoias.com |www.youtube.com/neoias|www.facebook.com/neoias | www.twitter.com/neoias

https://t.me/testseries4exams
Download All PDF form here :- https://t.me/pdf4exams
NEO IAS Most Important 101 History Model Questions from TPS 2018

Answer (c)
Explanation
The Sangam Age constitutes an important chapter in the history of South India.
According to Tamil legends, there existed three Sangams (Academy of Tamil poets) in
ancient Tamil Nadu popularly called Muchchangam. These Sangams flourished under
the royal patronage of the Pandyas. The first Sangam, held at Then Madurai, was
attended by gods and legendary sages but no literary work of this Sangam was available.
The second Sangam was held at Kapadapuram but the all the literary works had perished
except Tolkappiyam. The third Sangam at Madurai was founded by Mudathirumaran.
It was attended by a large number of poets who produced voluminous literature but only
a few had survived. These Tamil literary works remain useful sources to reconstruct the
history of the Sangam Age.

There is a plenty of information in the Sangam literature to trace the position of women
during the Sangam age. Women poets like Avvaiyar, Nachchellaiyar, and
Kakkaipadiniyar flourished in this period and contributed to Tamil literature. The
courage of women was also appreciated in many poems. Women were allowed to choose
their life partners. However, the life of widows was miserable. The practice of Sati was
also prevalent in the higher strata of society. The class of dancers was patronized by
the kings and nobles. The Hero Stone or Nadu Kal worship was significant in the
Sangam period. The Hero Stone was erected in memory of the bravery shown by the
warrior in battle. This kind of worshipping the deceased has a great antiquity.

50. The King kanishka held the great 4th Buddhist Council at kundalvana,Kashmir in
72AD.The deputy chairman of the council was?
(a) Nagasena
(b) Asvaghosha
(c) vasumitra
(d) (c)Upagupta

Answer (b)
Explanation
FOURTH COUNCIL
The fourth council The Fourth Buddhist Council was held at Kundalvana, Kashmir in
72 AD under the patronage of Kushan king Kanishka and the president of this council
was Vasumitra, with Aśvaghosa as his deputy. This council distinctly divided the
Buddhism into 2 sects Mahayana & Hinayana.

51. Consider the following pairs:


Saint Philosophy/school of=thought
1. Nagarjuna : Shunyavada
2. Ramanuja : Anekantavada
3. Shankaracharya : Advaita
Select the correct answer using the code given below.
(a) 1 only (b) 2 and 3 (c) 1 and 3 (d) 1, 2 and 3

52. With reference to Kautilyas Arthashastra, consider the following statements:


1. It was written in Prakrit language
2. It speaks the methods for screening ministers
3. it discusses a mixed economy
Select the correct statement/statements using the codes given below.
(a) 1 only (b) 2 only (c) 1 and 3 only (d) 2 and 3 only

NEO IAS  0484-4030104, 9446331522 Page 22


www.neoias.com |www.youtube.com/neoias|www.facebook.com/neoias | www.twitter.com/neoias

https://t.me/testseries4exams
Download All PDF form here :- https://t.me/pdf4exams
NEO IAS Most Important 101 History Model Questions from TPS 2018

Answer(d)
Explanation
KAUTILYA’S ARTHSHASHTRA
Arthshashtra was written in Sanskrit by Chanakya / Kautilya / Vishnugupta / Indian
Machiavelli. t includes books on the nature of government, law, civil and criminal
court systems, ethics, economics, markets and trade, the methods for screening
ministers, diplomacy, theories on war, nature of peace, and the duties and
obligations of a king.
The Arthashastra discusses a mixed economy, where private enterprise and state
enterprise frequently competed side by side, in agriculture, animal husbandry, forest
produce, mining, manufacturing and trade. However, royal statutes and officials
regulated private economic activities, some economic activity was the monopoly of
the state, and a superintendent oversaw that both private and state-owned enterprises
followed the same regulations. The private enterprises were taxed.
53. The defeat of Harshavardhana by Pulakeshin II on the banks of Narmada river is
mentioned in
(a) Hathigumpa inscription (b) Sonpat inscriptions
(c) Allahabad inscription (d) Aihole inscriptions

Answer (d)
Explanation
Pulakeshin II was the greatest ruler of the Chalukyas. He consolidated his authority in
Maharashtra and conquered large parts of Deccan. He defeated Harshavardhana in
circa AD 618 and acquired the title of dakshinapatheshvara (lord of the south).Aihole
inscription of Pulakeshin II mentioned his victory over Harshavardhana. Ravikirti, the
court poet of Pulakesin II composed the Aihole inscription.

54. Which of the following statements is/are not correct?


1. The Gomateshwara statue at Sravanabelagola represents son of second Tirthankara
2. Amaravati school of art sacked by Satvahanas
3. The Bharhut stupa was built by Ashoka
4. Pana, formed the imperial currency of the Mauryas
Select the correct statement/statements using the codes given below.
(a) 1 and 2 only (b) 3 and 4 only (c) 2 and 4 only (d) 1, 2 and 4 only

Answer (a)
Explanation
Gommateshwara Statue is a monolithic statue located at Shravanbelagola in the Indian
state of Karnataka.The statue Gommateshwara is dedicated to Bahubali,the son of
first thirthankara. It is one of the largest free standing statues in the world.
The Amravati school of art flourished in the region of Andhra Pradesh between the
lower valleys of rivers Krishna and Godavari. The main patrons of this art form were the
Satavahans but it carried on even later, patronized by their successor Ikshavaku rulers.
This art is said to have flourished between 150 BC and 350 AD. The Bharhut stupa was
first built by the Maurya king Ashoka in the 3rd century BCE, but many works of art,
particularly the gateway and railings, were apparently added during the Shunga period.
The use of currency, which began in the earlier period, became a common feature of
the Maurya period because of the developed commerce. Money was not only used for
trade; even the government paid its officers in cash. It seems that the punch-marked
silver coins, which carry the symbols of the peacock, and the hill and crescent, called
pana, formed the imperial currency of the Mauryas.

NEO IAS  0484-4030104, 9446331522 Page 23


www.neoias.com |www.youtube.com/neoias|www.facebook.com/neoias | www.twitter.com/neoias

https://t.me/testseries4exams
Download All PDF form here :- https://t.me/pdf4exams
NEO IAS Most Important 101 History Model Questions from TPS 2018

55. Consider the following statement :


1. James Prinsep, an officer in the mint of the east India Company, deciphered Brahmi
and Kharosthi scripts which were used in the earliest inscriptions and coins
2. James Prinsep found that most of the scripts mentioned a king referred to as
Piyadassi – meaning ‘pleasant to behold’
Which of the statements given above is/ are correct?
(a) 1, 2 and 4 (b) 1 and 2 only (c) 3 and 4 (d) 1 only

56. Consider the following statements


1. Chalcolithic people used painted pottery
2. Malwa (MP) and Jorwe (Nasik) are some Chalcolithic sites
3. They never practiced cultivation
Select the correct statement/statements using the codes given below.
(a) 1 only (b) 2 only (c) 1 and 2 only (d) 1, 2 and 3

Answer (c)
Explanation
Farming Communities, Kayatha (Ujjain, MP)
Animals domesticated were Cows, Ahar (Rajasthan) Malwa
CHALCOLITHIC Goats, Buffaloes, Sheep, And Pigs (M.P) Jorwe (Nasik)
(COPPER- Micro Lithic Tools of Stone and Copper,
STONE) Painted Pottery Black-and-Red
Pottery, No Knowledge of mixing two
Metals, not aware of the Art of
Writing

57. Consider the following statements about Ibn Battuta’.


1. He was a Persian traveller, wrote “Rihla”, in Persian
2. Muhammad Bin Tughlaq appointed him as ‘qazi or judge of Delhi.
Select the correct answer using the code given below:
(a) 1 only (b) 2 only (c) Both 1 and 2 (d) Neither 1 nor 2

Answer (b)
Explanation
Ibn Battuta was a medieval Moroccan traveller and scholar, who is widely recognized as
one of the greatest travellers of all time. He is known for his extensive travels, accounts
of which were published in his Travels (Rihla) which was written in Arabic. Over a period
of thirty years, Ibn Battuta visited most of the known Islamic world as well as many non-
Muslim lands. His journeys included trips to North Africa, the Horn of Africa, West Africa,
the Middle East, India, Central Asia, Southeast Asia, and China. Muhammad Bin
Tughlaq appointed him as ‘qazi or judge of Delhi as he was impressed by his scholarship.
Battuta was ordered in 1342 to proceed to China as the Sultan’s envoy to the Mongol
ruler.

58. Which of the following pairs are correctly matched?


(Bhakti saint) (place)
1. Tukram 1. Maharashtra
2. Surdas 2. Uttar Pradesh
3. Qutbuddin Bhaktiyar kaki 3. Delhi
4. Khwaja Muinuddin Chishti 4. Punjab
Select the correct answer using the codes given below.
(a) 1 and 2 only (b) 2, 3 and 4 only (c) 3 and 4 only (d) 1, 2 and 3 only

NEO IAS  0484-4030104, 9446331522 Page 24


www.neoias.com |www.youtube.com/neoias|www.facebook.com/neoias | www.twitter.com/neoias

https://t.me/testseries4exams
Download All PDF form here :- https://t.me/pdf4exams
NEO IAS Most Important 101 History Model Questions from TPS 2018

Answer (d)
Explanation
Tukram was a Bhakti saint from Mhaharashtra and contemporary of Sivaji and a great
devotee of Vithal of Pandharpur(Maharashtra). Also, he was responsible for creating a
background for Maratha nationalism.
Surdas (1483-1563) born in UP. He was the disciple of Vallabhacharya of south India.
He popularized the Krishna cult by singing songs glorifying Krishna’s childhood. He was
the author of the Sur Sagar, Sur Sarawali etc. Qutbuddin Bakhtiar Kaki was a
Muslim Sufi mystic, saint and scholar of the Chishti Order from Delhi, in what is now
India. He was the disciple and the spiritual successor of Moinuddin Chishti as head of
the Chishti order, and the person to whom the Qutb Minar, Delhi is dedicated. Khwaja
Moinuddin Chishti, was a Sufi, preacher, ascetic, religious scholar,
and philosopher from Sistan (Afghanisthan), who eventually ended up settling in
the Rajasthan and Ajmer was his main centre of work and he established the
famous Chishti Order of Sufism.

59. Allahabad Pillar inscription


1. Listed the achievements of Samudragupta
2. was issued in the language of Prakrit
3. was composed by Harisena,the court poet of Samudragupta
4. contained the inscriptions of Jahangir, the Mughal emperor
Select the correct answer using the codes given below.
(a) 1 and 2 only (b) 3 and 4 only (c) 1 and 3 only (d) 1, 3 and 4 only

Answer (d)
Explanation
Allahabad Pillar Inscription (Prayag Prasasti) This was issued by Samudragupta and
was composed by Harisena. It is written in very simple and refined Sanskrit in
Champu kavya style. It lists achievements of Samudragupta. This Inscription is a
eulogy of Samudragupta and mentions about the conquests of Samudragupta and
boundaries of the Gupta Empire. It also mentions that more than five states in the
frontier states surrendered and accepted his suzerainty. He had close contact with the
kingdom of Ceylon and South East Asian colonies. The eulogy of Harisena describes
him as hero of 100 battles. Also engraved on the stone are inscriptions by
the Mughal emperor, Jahangir, from the 17th century and Asoka of Mauryan
Empire.

60. Consider the following statement:


Mahavira:
1. regarded only animate objects are endowed with various degrees of consciousness.
2. objected to the Vedic rituals and Brahmin supremacy.
3. advocated that God had created world and exercises control over it.
4. advocated severe asceticism and extreme penance for the attainment of the highest
spiritual state.
Select the answer using the codes given below
(a) 1 and 3 only (b) 2 and 4 only (c) 2, 3, and 4 only (d) 1 and 4 only
Answer (b)
Explanation
Mahavira:
 Regarded all animate and inanimate objects are endowed with various degrees of
consciousness.
 objected to the Vedic rituals and Brahmin supremacy.
NEO IAS  0484-4030104, 9446331522 Page 25
www.neoias.com |www.youtube.com/neoias|www.facebook.com/neoias | www.twitter.com/neoias

https://t.me/testseries4exams
Download All PDF form here :- https://t.me/pdf4exams
NEO IAS Most Important 101 History Model Questions from TPS 2018

 advocated that world is created and sustained by a universal law


 Advocated severe asceticism and extreme penance for the attainment of the highest
spiritual state.
61. Which one of the following best describes the `Pavarana’ ceremony in Buddhism?
(a) The entrance exam ceremony for admission in to the Sangha.
(a) The ceremony performed by the monks for liberation of soul from karma
(b) The ceremony for expelling members of the sangha on disciplinary grounds.
(c) The ceremony of monks to confess their offences, if any, committed during their stay
at a fixed abode in the rainy season.
Answer (d)
Explanation
The `Pavarana’ ceremony in Buddhism is the ceremony of monks to confess their
offences, if any, committed during their stay at a fixed abode in the rainy season.
62. Which one among the following statements about the coins of the Gupta rulers is
correct?
(a) The obverse generally had king’s portrait and reverse always had animal motif
(b) The obverse generally had queen’s portrait and reverse always had a deity
(c) The obverse generally had king’s portrait and reverse always had spoked wheel
(d) The obverse generally had king’s portrait and reverse had an image of a deity

Answer (d)
Explanation
 The Gupta monarchs were famous for their gold coins.
 They also issued silver coins. However, coins made of copper, bronze or any other alloy
metals are scarce.
 The abundance of gold coins from the Gupta era has led some scholars to regard
this phenomenon as the ‘rain of gold’.
 The Gupta gold coin is known as dinaras. The gold coins of the Gupta rulers are the
extraordinary examples of artistic excellence.
 The coins depicted the ruling monarch on the obverse and carried legends with the
figure of a goddess on the reverse.

63. Which among the following statements is/are not correct?


1. The Madhuben plate inscription illustrated the chronology of Harshavardhana.
2. Harshacharita was written by his court poet Skanda Gupta.
3. Si-Yu-Ki, of Itsing threw light on the life and times of Harshavardhana
4. Harshavardhana was defeated by Pulakesin I on the banks of river Narmada
Select the correct answer using the code given below.
(a) 1 only (b) 4 only (c) 1, 2 and 4 only (d) 2, 3 and 4 only

Answer (d)
Explanation
HARSHAVARDHANA
 Harshavardhana then ascended the throne in AD 606. He was only sixteen years of
age at that time. Still he proved himself to be a great warrior and an able
administrator.
 There are two valuable sources that throw important light on the life and times of
Harshavardhana (606–647). These are Harshacharita written by his court poet
Banabhatta and Si-Yu-Ki, the travel account of the Chinese Buddhist pilgrim Hsuan
Tsang, who visited India during AD 629–644.

NEO IAS  0484-4030104, 9446331522 Page 26


www.neoias.com |www.youtube.com/neoias|www.facebook.com/neoias | www.twitter.com/neoias

https://t.me/testseries4exams
Download All PDF form here :- https://t.me/pdf4exams
NEO IAS Most Important 101 History Model Questions from TPS 2018

 The Madhuben plate inscription and the Sonpat inscription are also helpful to know
the chronology of Harsha.
 Harsha wanted to extend his power in the Deccan. But he was defeated by Pulakesin
II, the Chalukya ruler, on the banks of river Narmada.
64. Consider the following statements:
1. Diwan-i-riyasat was devised for market regulations and controlling prices.
2. Diwan-i-Khairat was a department of slaves started by Firuz Shah Tughlaq
3. Diwan- I-Kohi was a department of currency exchange, created by Muhammad bin
Tughluq.
Which of the statements given above is/are correct?
(a) 1 only (b) 2 only (c) 1 and 2 only (d) 2 and 3 only

Answer (a)
Explanation
 Alauddin Khilji, created Diwan-i-riyasat whose primary function was to implement the
economic regulations issued by the Sultan and control the markets and prices.
 Diwan-i-bandagan (department of slaves) and Diwan-i-Khairat (charity department)
was created by Firuz shah Tughluq.
 Diwan-i-mustakharaj (to realise arrears) was created by Alauddin Khiiji.
 Diwan- i-kohi (department of agriculture) was created by Muhammad bin Tughluq.

65. Consider the following statements about Suhrawardi silsilah:


1. Sheikh Bahauddin Zakaria established it in India.
2. They led a life of poverty and simplicity.
3. They established themselves mainly in Punjab and Sind.
4. Sheikh Qutbuddin Bakhtiyar Kaki was a follower of this order.
Which of the statements given above is/are correct?
(a) 1 and 3 only (b) 3 and 4 only (c) 2 and 4 only (d) 1, 2 and 4 only

Answer (a)
Explanation
SUHRAWARDI ORDER
 Founder — Shihabuddin Suhrawardi (12th & 13th centuries)
 Other leaders — Bahauddin Zakariya (13th century), Ruknuddin Abdul Fath (14th
century), etc.
 Established themselves mainly in north western India.
 They did not believe in leading a life of poverty and so accepted the direct service
of the state.
66. The Battle of Chandawar, fought in last decade of 12th century, was between
(a) Prithviraj Chauhan and Bhimdev Solanki
(b) Prithviraj Chauhan and Mohammad Ghori
(c) Jaichandra and Mohammad Ghori
(d) Jaichandra and Prithviraj Chauhan
Answer (c)
Explanation
 Jayachandra, the last important Gahadvala, was defeated by Mohammad Ghori in
the battle of Chandwar (1193).
67. Which one of the following is correct in context to Kalabhra Interregnum?
(a) Kalabhra were vanquished by Sangam Cholas
(b) They practiced Brahmadeya
NEO IAS  0484-4030104, 9446331522 Page 27
www.neoias.com |www.youtube.com/neoias|www.facebook.com/neoias | www.twitter.com/neoias

https://t.me/testseries4exams
Download All PDF form here :- https://t.me/pdf4exams
NEO IAS Most Important 101 History Model Questions from TPS 2018

(c) They ruled Tamil kingdom for four decades


(d) They patronized Buddhism and Jainism

Answer (d)
Explanation
 The Kalabhra dynasty ruled over the entire ancient Tamil country between the 3rd
and the 7th century in an era of South Indian history called the Kalabhra interregnum.
 The Kalabhras, displaced the Sangam kingdoms of the Cholas, Pandyas and Chera
dynasties by a revolt.
 Later, The Kalabhras were defeated by the joint efforts of the Pallavas, Pandyas
and Chalukyas of Badami.
 They are generally believed to have been hill tribes that rose out of obscurity to become
a power in South India. Their kings were likely followers of Buddhism and Jainism.
 Buddhadatta, the great writer in Pali, belonged to Uraiyur. He mentions his
contemporary, King Achyutavikranta of the Kalabharakula, as ruling over the Chola
country from Kaveripumpattinam. He was a Buddhist.
 Tamil literary tradition refers to an Achyuta who kept the Chera, Chola and Pandya
rulers in captivity.
68. Which one of the following is the correct chronological order?
(a) Mahmud of Ghazni — Muhammad Ghori –– Qutubuddin Aibak— Muhammad bin
Qasim
(b) Muhammad bin Qasim — Qutubuddin Aibak — Mahmud of Ghazni — Muhammad
Ghori
(c) Muhammad Ghori — Mahmud of Ghazni — Qutubuddin Aibak — Muhammad bin
Qasim
(d) Muhammad bin Qasim--- Mahmud of Ghazni — Muhammad Ghori — Qutubuddin
Aibak
Answer (d)
Explanation
 Muhammad bin Qasim (a deputy of the Arab governor of Iraq, Hajjaj) invaded and
occupied Sind in 712 AD from its Hindu ruler, Dahir. Henceforth Sind continued to
be under Muslim occupation.
 Mahmud of Ghazni invaded India from 997 to his death in 1030
 Muhammad Ghori who invaded India after Mahmud Ghazni, belongs to Ghurid
empire and attempted to build up an empire in India
 Qutubuddin Aibak, who founded slave dynasty in India continued from 1206-1210
AD
 Ghiyasuddin Balban, the fourth important ruler of slave dynasty continued from
1266-1287
69. With reference to the reforms of Alauddin Khalji, which of the following statements
is/are not correct ?
1. Resumption of land grants such as ‘inam’, ‘waqf’.
2. Ban on intermarriages and gathering among nobles without consent of Sultan.
3. Removal of the ban on public sale of liquor and drugs.
Select the correct answer using the code given below.
(a) 1 and 3 only (b) 2 only (c) 3 only (d) 2 and 3 only

NEO IAS  0484-4030104, 9446331522 Page 28


www.neoias.com |www.youtube.com/neoias|www.facebook.com/neoias | www.twitter.com/neoias

https://t.me/testseries4exams
Download All PDF form here :- https://t.me/pdf4exams
NEO IAS Most Important 101 History Model Questions from TPS 2018

Answer (c)
Explanation
REFORMS OF ALAUDDIN KHALJI
 Confiscation of all grants of tax- free land;
 Reorganisation of the espionage system;
 Prohibition of public sale and consumption of liquor and drugs; and
 Ban on social gatherings of nobles and inter-marriages among them without prior
permission of the Sultan.
MILITARY REFORMS:
 Introduction of ‘dagh’ (branding of horses) and ‘chahra’ (descriptive roll of
soldiers).
 Insistence on regular muster of the army.
 Abolition of the ‘iqtas’ (areas whose revenues were assigned to soldiers in lieu of their
salaries) of the royal troopers and payment of their salaries in cash (iqtas of big
nobles and military commanders were, however, allowed to continue).
ECONOMIC REFORMS:
 Establishment of separate markets for food-grains, cloth, horses, fruits, etc. and
their strict regulation.
 Increase of land revenue to 50 per cent of the gross produce and elimination of all
middlemen.
 Resumption of several types of land grants such as ‘inam’, ‘waqf’, etc.
 Appropriation of 4/5th share of the war booty (khums) to the state leaving only 1/5th
to the soldiers.
 Creation of a new department, viz ‘diwan-i-mustahraj’, to enquire into the revenue
arrears and to collect them.
70. With reference to the cultural history of India, consider the following statements:
1. Kalbelia is folk songs and dances of Manipur.
2. The Kumbha Mela festival is held in Allahabad, Haridwar, Ujjain and Nasik
3. Ramman is a religious festival and ritual theatre of Rajasthan
Which of the statements given above is/ are correct?
(a) 1, 2 and 3 (b) 2 only (c) 3 only (d) 1 and 3 only

Answer (b)
Explanation
KUMBH MELA
KumbhMela is held every four times every 12 years at four different locations across
central and northern India. It is the largest religious congregation and largest peaceful
gathering on planet. The festival, held in Allahabad, Haridwar, Ujjain and Nasik,
represents a syncretic set of rituals related to worship and ritual cleansing in holy rivers
in India.
UNESCO’s List of Intangible Cultural Heritage of Humanity from India
 Koodiyattam: Sanskrit Theatre of Kerala.
 Mudiyett: theatre ritual of Kerala.
 Tradition of Vedic Chanting.
 Kalbelia: folk songs and dances of Rajasthan.
 Ramlila: Traditional Performance of the Ramayana.
 Sankirtana: singing, drumming and dancing ritual of Manipur.
 Ramman: religious festival and ritual theatre of Garhwal Himalayas.
 Traditional brass and copper craft of utensil of Thatheras: Punjab.
 Chhau dance: classical Indian dance originated in the eastern Indian states.

NEO IAS  0484-4030104, 9446331522 Page 29


www.neoias.com |www.youtube.com/neoias|www.facebook.com/neoias | www.twitter.com/neoias

https://t.me/testseries4exams
Download All PDF form here :- https://t.me/pdf4exams
NEO IAS Most Important 101 History Model Questions from TPS 2018

 Buddhist chanting of Ladakh: recitation of sacred Buddhist texts in Ladakh region of


Jammu and Kashmir
 Yoga
 Kumbh Mela
71. Consider the following pairs:
1. Rani ki Vav : Udayamati
2. Battle of Tarain (1191) : Victory of Muhammad Ghori
3. Vikramsila University : Dharma Pala
4. Turushkadanda : Property Tax
Which of the pairs given above is/are not correctly matched?
(a) 1, 2 and 3 only (b) 2 and 3 only (c) 3 and 4 only (d) 2 and 4 only

Answer (d)
Explanation
 The Rani ki Vav (the Queen’s Stepwell) was a stepwell built by Queen Udaymati as a
memorial to her husband Bhimdev I, the Solanki king in the 11th century AD and is
located in the north Gujarat district of Patan on, the banks of Saraswati. It was
added to the UNESCO’s World Heritage List in 2014.
 GAHADVALAS:
With their capital at Kanauj, they came into prominence under Chandradeva, who
imposed a tax called ‘Turushkadanda’ either to meet the expenses of war against
Muslim invaders or to pay a tribute to the latter.
 Vikramasila University was founded by Dharmapala of Pala dynasty (8th 9th
centuries)
 The first battle of tarain,1191. Muhammad Ghori was defeated by the Rajput force
under Prithviraj chauhan
72. Consider the following statements about Silambam
1. It is a traditional martial art form of Manipur
2. It was patronized by the Chera, Chola and Pandya rulers of Sangam age
Which of the statements given above is/ are correct?
(a) 1 only (b) 2 only (c) Both 1 and 2 (d) Neither 1 nor 2

Answer (b)
Explanation
 Silambam is an ancient stick-martial art of Tamil Nadu.
 According to Sangam-era literature, there were a number of exercise centres called
silambak-koodams all over the Dravida Nadu region, and the age-old art—patronized
by the Chera, Chola and Pandya rulers—is recorded as one of the 64 art forms of
ancient India.
73. With reference to the cultural history of India, a style of temple construction in which
main shrine is surrounded by four supplementary shrines is known as
(a) Nanchayatan Style
(b) Punchamachetan Style
(c) Ahraharam Style
(d) Panchayatan Style
Answer (d)
Explanation
A style of temple construction in which main shrine is surrounded by four
supplementary shrines is known as

NEO IAS  0484-4030104, 9446331522 Page 30


www.neoias.com |www.youtube.com/neoias|www.facebook.com/neoias | www.twitter.com/neoias

https://t.me/testseries4exams
Download All PDF form here :- https://t.me/pdf4exams
NEO IAS Most Important 101 History Model Questions from TPS 2018

74. Most of them have inscriptions in a pictographic script that is yet to be deciphered. The
script was written from right to left. Animal impressions were also carved on the surfaces.
The common animal motifs were unicorn, humped bull, rhinoceros, tiger, elephant,
buffalo, goat, etc. However, no evidence of cow has been found in any of them. This
description refers to
(a) the Indus Terracotta Images
(b) the Indus Coins
(c) the Indus Seals
(d) the Indus Dice
Answer (c)

75. Consider the following statements:


1. An important characteristic of the Mathura school is the ‘narrative art’.
2. Earlier Mathura sculptures hardly had Greek influences in the depiction Buddha.
3. The main theme of Gandhara school can be said to be the new form of Buddhism, viz.
Hinayanism.
Which of the statements given above is/are correct?
(a) 1 and 2 only (b) 1, 3 only (c) 2 only (d) 1, 2 and 3
Answer (a)
Explanation
 An important characteristic of the Mathura school is the ‘narrative art’.
 Earlier Mathura sculptures hardly had Greek influences in the depiction Buddha.
 The main theme of Gandhara school can be said to be the new form of Buddhism, viz.
Mahayanism.
76. Consider the following statements
1. Indo Islamic architecture is a fusion of Persian Arabic Turkish and Indian styles
2. It is known for its intricate jaali works and use of lintel and beams
Select the correct answer using the codes given below
(a) 1 only (b) 2 only (c) Both 1and 2 (d) Neither 1 nor 2
Answer (a)
Explanation
 Indo Islamic architecture is a fusion of Persian Arabic Turkish and Indian styles
 It is known for its intricate jaali works and but no use of lintel and beams
77. The causes of the decline of Buddhism did not include
(a) Division of Buddhism
(b) Loss of royal patronage
(c) Adoption of Sanskrit
(d) Rejection of divinity of Vedas
Answer (d)
Explanation
Rejection of divinity of Vedas was a reason for the popularity of the unorthodox religions
like Buddhism Jainism in 6th BC
78. The theme of the murals of the Ajanta caves are
1. Geometrical designs of ancient stupas
2. Portraits of various Buddha’s and Bodhisattvas
3. Narrative scenes portraying Jatakas stories
Select the correct answer using the code given below:
(a) 1, 2 and 3 (b) 2 and 3 (c) 1 and 3 (d) 1 and 2
NEO IAS  0484-4030104, 9446331522 Page 31
www.neoias.com |www.youtube.com/neoias|www.facebook.com/neoias | www.twitter.com/neoias

https://t.me/testseries4exams
Download All PDF form here :- https://t.me/pdf4exams
NEO IAS Most Important 101 History Model Questions from TPS 2018

Answer (b)
Explanation
The theme of the murals of the Ajanta caves are Portraits of various Buddha’s and
Bodhisattvas & Narrative scenes portraying Jatakas stories.

79. Which one among the following pairs is not properly matched?
(a) Megasthenes : Indica
(b) Ashvaghosa : Buddhacharita
(c) Panini : Mahabhashya
(d) Vishakhadatta : Mudrarakshasa
Answer (c)
Explanation
Mahabhashya was written by Patanjali
80. Which one of the following temple structure of India does represent the typical Vesara
style?
(a) Brihadeswara temple at Tanjore
(b) Kandariya Mahadev temple at Khajuraho
(c) The Channa Kesava temple at Belur
(d) Modhera Sun temple at Mehsana
Answer (c)
Explanation
Kandariya Mahadev temple at Khajuraho represents the typical Nagara style whereas
the Channa Kesava temple at Belur, Hoyasalesvara temple at Halebid represent Vesara
style and Brihadeswara temple at Tanjore represents Dravidian style.
81. Which one the following Act appointed Warren Hastings as the first Governor-General of
the Presidency of Fort William?
(a) The Regulating Act of 1773
(b) The Saint Helena Act of 1833
(c) The Pitts India Act of 1784
(d) The Charter Act of 1813

NEO IAS  0484-4030104, 9446331522 Page 32


www.neoias.com |www.youtube.com/neoias|www.facebook.com/neoias | www.twitter.com/neoias

https://t.me/testseries4exams
Download All PDF form here :- https://t.me/pdf4exams
NEO IAS Most Important 101 History Model Questions from TPS 2018

Answer (a)
Explanation
 The Regulating Act of 1773 created the office with the title of Governor-General of the
Presidency of Fort William, or Governor-General of Bengal to be appointed by the
Court of Directors of the East India Company (EIC). Warren Hastings became first
Governor General of Bengal.
 The Saint Helena Act 1833 (or the Charter Act of 1833) re-designated the office with
the title of Governor-General of India. Lord William Bentick (1828-1835) became
the first Governor General of India.
82. With reference to ‘Vijayanagara temple architecture’, consider the following statements:
1. Raya gopurams were high entrance gateways to the temples.
2. Pillars are ornated by the figures of horses in jumping postures.
Which of the statements given above is/are correct?
(a) 1 only (b) 2 only (c) Both 1 and 2 (d) Neither 1 nor 2

Answer (c)
Explanation
 Raya gopurams were high entrance gateways to the temples
 Pillars are mostly ornated by the figures of horses in jumping postures

83. What was the main difference between the Indus civilization and Vedic civilization?
(a) Indus civilization was urban whereas the Vedic civilization was rural.
(b) The Indus civilization more relied on trade, whereas for Vedic age, it was on
agriculture.
(c) The Vedic religion was monotheistic whereas it was polytheistic in Indus civilization
(d) The inscriptions of Vedic culture was alphabetic unlike the pictographical script of
Indus people
Answer (a)
Explanation
 Indus civilization was urban whereas the Vedic civilization was rural.
 The Indus civilization more relied on trade, whereas in the Vedic age, it was mainly
pastoral.
 Both civilization had polytheistic religion
 The Vedic people know no art of writing whereas Indus people had the pictographical
script.
84. Consider the following statement
1. Shishunaga’s greatest achievement was the destruction of the power of Avanti.
2. Mahapadma Nanda took the title of Ekarat.
Which of the statements given above is/are correct?
(a) 1 only (b) 2 only (c) Both 1 and 2 (d) Neither 1 nor 2

Answer (c)
Explanation
 Shishunaga’s greatest achievement was the destruction of the power of Avanti.
 Mahapadma Nanda took the title of Ekarat
85. With reference to the religious history of India, consider the following statements:
1. Bodhisattvas were non-living entities.
2. The concept of Bodhisattva is common in all forms of Buddhism.
Which of the statements given above is/are correct?
(a) 1 only (b) 2 only (c) Both 1 and 2 (d) Neither 1 nor 2
NEO IAS  0484-4030104, 9446331522 Page 33
www.neoias.com |www.youtube.com/neoias|www.facebook.com/neoias | www.twitter.com/neoias

https://t.me/testseries4exams
Download All PDF form here :- https://t.me/pdf4exams
NEO IAS Most Important 101 History Model Questions from TPS 2018

Answer (d)
Explanation
 A bodhisattva is literally a living being (sattva) who aspires to enlightenment (Bodhi)
and carries out altruistic practices.
 The bodhisattva ideal is central to the Mahayana Buddhist tradition as the individual
who seeks enlightenment both for himself/herself and for others. Compassion, an
empathetic sharing of the sufferings of others, is the bodhisattva's greatest
characteristic.
 A bodhisattva is a being who carries out the work of the Buddha’s, vowing not to
personally settle into the salvation of final Buddhahood until she or he can assist
all beings throughout the vast reaches of time and space to fully be free.

86. Which of the following statement/statements is/are correct about Buddhist Sangha?
1. Sutta Pitaka contain the injunction made for the Buddhist sangha
2. The Sangha excluded brahmins, kshatriyas, merchants and debtors
Select the correct answer using the code given below.
(a) 1 only (b) 2 only (c) Both 1 and 2 (d) Neither 1 nor 2

Answer (d)
Explanation
 The rules made for the Buddhist sangha were written down in a book called the
Vinaya Pitaka. Those who worked for the king, like soldiers, had to take his
permission and debtors that of creditors. Otherwise they will not be permitted to
become members of sangha.
 Women had to take their husbands’ permission. Children had to take the permission
of their parents and slaves that of their masters.
 Men and women who joined the sangha led simple lives. Those who joined the
sangha included brahmins, kshatriyas, merchants, labourers, barbers, courtesans
and slaves.
87. Tribhanga is mostly associated with
(a) Bharatanatyam
(b) Odissi
(c) Kathakali
(d) Kuchipudi

Answer (b)
Explanation
 The tribhanga posture, i.e. the three bended form of the body is innate to Odissi
dance form.
 Odissi dance form is unique in its representation of gracefulness, sensuality and
beauty. The dancers create intricate geometrical shapes and patterns with her body.
Hence, it is known as ‘mobile sculpture’.
88. Which of the following was not found at Lothal?
(a) Rice husk
(b) Artificial dockyard
(c) Iranian seal
(d) a terracotta plough

NEO IAS  0484-4030104, 9446331522 Page 34


www.neoias.com |www.youtube.com/neoias|www.facebook.com/neoias | www.twitter.com/neoias

https://t.me/testseries4exams
Download All PDF form here :- https://t.me/pdf4exams
NEO IAS Most Important 101 History Model Questions from TPS 2018

Answer (d)
Explanation
 Important Findings of Lothal an artificial dockyard {which makes it an important
sea link}, rice husk {rice husk has been found only at Lothal and Rangpur}, bead
making factory etc.
 Lothal is thought to have direct sea trade links with Mesopotamia because of finding
of an Iranian seal from there. Terracotta plough is from Banawali.
89. With reference to Ancient India, consider the following statements
1. The Junagarh inscription describes the rule of Indo-Greeks or Bactrians.
2. Milindpanho is the dialogues between Menander, Bactrian king and Nagsena, a
monk.
3. Rudradaman I was a Shaka ruler, who won the title of Mahakshatrapa.
Which of the statements given above is/are correct?
(a) 1 only (b) 2 and 3 only (c) 1 and 3 only (d) 1, 2 and 3

Answer (b)
Explanation
 The second and third statements are correct
 The Junagarh inscription near Kathiawar mentions the rule of Shaka rulers.
 The second and third statements are self explanatory.
90. With reference to Ancient India, which of the following pairs is/are correctly matched?
1. Satavahanas - Pratisthana
2. Bactrians - Sakala
3. Shakas - Taxila
Select the correct answer using the code given below:
(a) 1 and 2 only (b) 2 and 3 only (c) 1 and 3 only (d) 1, 2, and 3

Answer (d)
Explanation
 All the statements are correct.
 The places are capitals of the respective dynasties.
 Pratisthana is modern day Paithan. Sakala is modern Sialcot.
91. With reference to Sufism in medieval India, consider the following:
Sufis followed
1. Performance of prayers and pilgrimages
2. Charity and suppression of passions
3. Strict observance of the Sharia
4. Self-discipline
Which of the statements given above is / are correct?
(a) 1 only (b) 2 only (c) 2 and 3 only (d) 1, 2 and 4 only
Answer (d)
Explanation
Sufism promotes tolerance and acceptance of other religions and points of view. Sufism
stressed the elements of love and devotion as effective means of the realization of God.
Love of God meant love of humanity and so the Sufis believed service to humanity was
equal to service to God. In Sufism, self-discipline was considered an essential condition
to gain knowledge of God by sense of perception. While orthodox Muslims emphasize
external conduct, the Sufis lay stress on inner purity. While the orthodox believe in blind
observance of rigid rituals, the Sufis consider love and devotion as the only means of
attaining salvation.
NEO IAS  0484-4030104, 9446331522 Page 35
www.neoias.com |www.youtube.com/neoias|www.facebook.com/neoias | www.twitter.com/neoias

https://t.me/testseries4exams
Download All PDF form here :- https://t.me/pdf4exams
NEO IAS Most Important 101 History Model Questions from TPS 2018

According to them one must have the guidance of a pir or guru, without which spiritual
development is impossible. Sufism also inculcated a spirit of tolerance among its
followers. Other ideas emphasized by Sufism are meditation, good actions, repentance
for sins, performance of prayers and pilgrimages, fasting, charity and suppression
of passions.

92. With reference to Kushana dynasty, which of the following statements is/are correct?
1. The Kushana dynasty was established by Kadphises I.
2. The title ‘Devaputra’, was used by Kushanas to claim divinity.
3. The ‘Rabatak’ inscription mentions about Kushana king, Vima Kadphises.
Select the correct answer using the code given below:
(a) 1 and 2 only (b) 2 and 3 only (c) 3 only (d) 1, 2 and 3
Answer (d)
Explanation
 All the statements given are correct.
The Rabatak inscription talks about Vima Kadphises who is the father of Kanishka.
93. With reference to trade in Post Mauryan period, which of the following statements
is/are correct?
1. Ujjain was a famed bead making centre.
2. Indo-Roman trade is described in the book, Periplus of Erythrean Sea.
Select the correct answer using the code given below:
(a) 1 only (b) 2 only (c) Both 1 and 2 (d) Neither 1 or 2
Answer (c)
Explanation
 Both the statements are correct.
 Ujjain was famous for its beads and Vanga for textiles.
 The author of Periplus of Erythrean Sea is unknown.
94. Consider the following statements regarding Kathak
1. It was patronized by Wajid Ali Shah
2. Its main centres are Jaipur, Banaras and Lucknow.
Which of the statements given above is/are correct?
(a) 1 only (b) 2 only (c) Both 1 and 2 (d) Neither 1 nor 2

95. With reference to trade during Mughal period, consider the following statements:
1. Seth, Bohra traders are specialized in long distance trade.
2. Local short distance traders were called Banjaras.
Which of the statements given above is/are correct?
(a) 1 only (b) 2 only (c) Both 1 and 2 (d) Neither 1 nor 2

Answer (a)
Explanation
Seth, Bohra traders specialized in long distance trade while local traders were called
Banik. Another class of traders was known as banjaras, who specialized in carrying bulk
goods. The banjaras used to move to long distances with their goods on the back of oxen.
Bulk goods were also taken through rivers on boats.
96. With reference to Mughal administration, which of these statement/statements is/are
correct?
1. Mansabdars were both civil and military officers in Mughal administration.
2. The army was headed by the King.

NEO IAS  0484-4030104, 9446331522 Page 36


www.neoias.com |www.youtube.com/neoias|www.facebook.com/neoias | www.twitter.com/neoias

https://t.me/testseries4exams
Download All PDF form here :- https://t.me/pdf4exams
NEO IAS Most Important 101 History Model Questions from TPS 2018

3. Provincial ‘diwan’ was the governor of the province.


Select the correct answer using the code given below.
(a) 1 and 2 only (b) 1 only (c) 2 and 3 only (d) 1, 2 and 3

Answer (a)
Explanation
 The first and second statement is correct.
 Subehdar is the governor of the 'subah' or province.
 Provincial diwan looks after the revenue administration of the province.

97. Battle of Chausa was fought between?


(a) Babur and Sher Shah
(b) Humayun and Sher shah
(c) Babur and Rana of Mewar
(d) Humayun and Muhammad shah of Gujarat
Answer (b)
Explanation
Battle of Chausa was fought between Humayun and Sher Shah which was held in 1539
and Sher Khan destroyed the Mughal army and Humayun escaped from there.
98. Which one of the following statements is not correct about Koodiyattam?
(a) It is based on Sanskrit theatre traditions.
(b) It is practiced by women folk of Tamil Nadu
(c) it is performed in Koothambalams
(d) it put emphasis on hand gestures and eye movements
Answer (b)
Explanation
it belongs to the state of Kerala

99. With reference to Bhand Pather, consider the following statements


1. It is a traditional theatre form of Kashmir
2. it is a unique combination of dance, music and acting
3. it begins with a ritualistic dance called chhok
Which of the statements given above is/are correct?
(a) 1, 2 and 3 (b) 1 and 3 only (c) 2 and 3only (d) 3 only
Answer (a)

100. Consider the following statements regarding Swang,the folk theatre:


1. It is a folk theatre form in Haryana, Rajasthan and Uttar Pradesh
2. It has two important styles,Rohtak and Haathras
3. Rohtak is performed in the Sanskrit language
Which of the statements given above is/are correct?
(a) 1 and 2 only (b) 2 and 3 only (c) 3 only (d) none

Answer (a)
Explanation
Swang has two important styles – one that belong to Rohtak (performed in the Bangru
language) and the other that belongs to Haathras (performed in the Brajbhasha
language).

NEO IAS  0484-4030104, 9446331522 Page 37


www.neoias.com |www.youtube.com/neoias|www.facebook.com/neoias | www.twitter.com/neoias

https://t.me/testseries4exams
Download All PDF form here :- https://t.me/pdf4exams
NEO IAS Most Important 101 History Model Questions from TPS 2018

101. Tahzibul Akhlaq was


(a) a journal for the social reformation of Muslims of India
(b) biography of sir Sayyid Ahmad khan
(c) newspaper published by Raja Ram Roy
(d) Book on Indian Islamic culture

Answer (a)
Explanation
Tehzeeb-ul-Ikhlaq was a journal published by the reformer Sir Syed Ahmed Khan
between 1871 and 1897. The journal published alternative Muslim perspectives, written
in plain language. It gave voice to the publisher's religious, social, and reforming
opinions, and is credited with establishing him as one of the fathers of Urdu fiction.

NEO IAS  0484-4030104, 9446331522 Page 38


www.neoias.com |www.youtube.com/neoias|www.facebook.com/neoias | www.twitter.com/neoias

https://t.me/testseries4exams
Download All PDF form here :- https://t.me/pdf4exams
NEO IAS Most Important 101 History Model Questions from TPS 2018

NEO IAS  0484-4030104, 9446331522 Page 39


www.neoias.com |www.youtube.com/neoias|www.facebook.com/neoias | www.twitter.com/neoias

https://t.me/testseries4exams

You might also like